jueves, 28 de mayo de 2015

Problema de Algebra (facil para empezar)

Sean $x$, $y$, $z$ numeros reales positivos tales que $x+y+z=1$. Muestra que
$$\frac{(1+xy+yz+zx)(1+3x^3+3y^3+3z^3)}{9(x+y)(y+z)(z+x)} \geq$$

$$\left(\frac{x\sqrt{1+x}}{\sqrt[4]{3+9x^2}} + \frac{y\sqrt{1+y}}{\sqrt[4]{3+9y^2}}+\frac{z\sqrt{1+z}}{\sqrt[4]{3+9z^2}}\right)^2.$$

8 comentarios:

Juan dijo...

Veamos que la parte de abajo tiene

$\sqrt{(1+1+1+(3x)^2)/4)} \ge (3/4)(1+x)$

y entonces cada factor es menor o igual a $ x/\sqrt{3/2}$ y te da que toda la suma es menor o igual a $ \sqrt{2/3}$.

Entonces hay que ver que la parte de arriba es mayor o igual a $2/3$. con $d=xy+yz+zx, X=xyz$ da que hay que demostrar $ X \ge \frac{9d^3+11d-4}{15+9d} $.

Veamos que $ P(t)=(t-x)(t-y)(t-z)=t^3-t+td-X $ tiene tres raíces reales positivas. $P'(t)=0$ con $t=(1+\sqrt{1-3d})/3$ o $t=(1-\sqrt{1-3d})/3$.

Entonces el primer $t$ debe satisfacer $t \le 0$, y esto da que con $t=(1+\sqrt{1-3d})/3$, tenemos

$X \ge t^3-t^2+td $.

y obtuve que $t^3-t^2+td \ge \frac{9d^2+11d-4}{15+9d} $ (desigualdad de una sola variable), pero no estoy 100% seguro que este bien.

Juan dijo...
Este comentario ha sido eliminado por el autor.
Juan dijo...
Este comentario ha sido eliminado por el autor.
Juan dijo...

Suponiendo $d \neq 1/3$ (si $d=1/3$ entonces es fácil), usando $t^2=(2t-d)/3$ llegas a que quieres demostrar $t \le 2(2x+3)/(3x+5)$, y eso se reduce a $\sqrt{1-3x} \le (9x+13)/(3x+5)$ lo cual es obvio pues la izquierda es <1 y la derecha es >1.

Juan dijo...

nota: $t$ es mayor valor tal que la derivada de P en $t$ es 0.

Juan dijo...

O sea, básicamente lo que hice para ver que la primera parte es >2/3 fue ver que en los puntos locales minimos y maximos de P, el segundo valor te da un P negativo, pues P tiene 3 raíces. además, P empieza en negativos y termina en positivos. Entonces obtenemos la desigualdad $P(t) \le 0$ y entonces $ X \ge t^3-t^2+td \ge \text{fraccion fea} $, y acabo.

Para ver que $t^3-t^2+td \ge \text{fraccion fea}$ utilice que $t^2=(2t-d)/3$ y se reduce a $t \le 2(2x+3)/(3x+5)$, lo cual se reduce a $\sqrt{1-3x} \le (9x+15)/(3x+5).$

nivek dijo...

yo empece igual que juan y quise ver que lo de arriba era mayor o igual a ${frac{2}{3}$ y para eso primero es obvio que $x^3+y^3+z^3 \geq \frac{1}{9}$ entonces queremos que $\frac{1+xy+yz+zx}{(1-x)(1-y)(1-z)} \geq \frac{9}{2}$ y esto se reduce a $1+\frac{9}{2}xyz-\frac{7}{2}(xy+yz+zx) \geq 0$ y esto es
$2(x^2+y^2+z^2)-3(xy+yz+zx)+9xyz \geq 0$ pero
$3(x^3+y^3+z^3)-9xyz=3(x^2+y^2+z^2-xy-yz-zx)$
entonces nuestra desigualdad es
$3(x^3+y^3+z^3)-(x^2+y^2+z^2) \geq 0$
pero utilizando MC-MQ-MA se puede ver que esto es cierto (con MC me refiero media cúbica)

nivek dijo...
Este comentario ha sido eliminado por el autor.

Publicar un comentario